Questions tagged [stochastic-calculus]

Stochastic calculus provides a consistent theory of integration for stochastic processes and is used to model random systems. Its applications range from statistical physics to quantitative finance.

Filter by
Sorted by
Tagged with
1 vote
0 answers
189 views

Stochastic volatility model question

Let suppose that $S_t$ is a process defined as: $$ \begin{cases}dS_t = \mu S_t\,dt+m(v_t)\,dW^1_t\\ dv_t = \mu_v(v_t)\,dt + \sigma_v(v_t)\,dW^2_t\end{cases}$$ where the two Brownian motions have ...
NancyBoy's user avatar
  • 175
6 votes
0 answers
226 views

Global well posedness of $\phi^4_1$

We consider the $\phi^4_1$ model: $\partial_t\phi=\Delta\phi-\phi^3+\xi$ on $[0,T] \times \mathbb{R},$ where $\xi$ is a space time white noise. I know how to solve this equation locally on the torus, ...
mathex's user avatar
  • 399
4 votes
1 answer
247 views

Reference request: showing that solution of an Ito SDE stays bounded with positive probability

Assume that we have a (well-posed) Ito SDE of the form $$\mathrm{d} X_t = b(X_t)\,\mathrm{d} t + \sigma(X_t)\,\mathrm{d}W_t \label{1}\tag{1},$$ where $b \colon \mathbb{R}^d \to \mathbb{R}^d$, $\sigma \...
Fei Cao's user avatar
  • 700
1 vote
0 answers
95 views

Freidlin Wentzell for stochastic differential inclusions

Consider the SDI $$dX^\varepsilon(t)\in b(X^\varepsilon(t))\,dt + \varepsilon \sigma(X^\varepsilon(t)) \, dB(t).$$ Is there any Freidlin-Wentzell large deviations principle for $X^\varepsilon$?
user479223's user avatar
  • 1,302
2 votes
0 answers
60 views

On the measurability of stochastic integrals

Let $S(t)$ be a $C_0$-contraction semi-group, $W$ is a cylindrical Wiener process in a separate Hilbert space $U$. Assume the following conditions: $$ \|F(t,u_1)-F(t,u_2)\|_{H}< C\|u_1-u_2\|_{H},~~...
ABCD's user avatar
  • 31
2 votes
0 answers
108 views

Controlling the adjoint variables in a stochastically perturbed control problem

Suppose we have a deterministic control problem $$dX_t = b(X_t, u_t) \, dt$$ on a finite timeframe with no terminal cost; i.e. the objective functional to be maximised is $$\mathbb E \left [\int_{0}^T ...
Nate River's user avatar
  • 4,832
1 vote
0 answers
79 views

SPDE via fixed point argument and Young's theorem

Let $(P_r)_{r\geq 0}$ be a strongly continuous semi-group (not necessarily the heat kernel). It is well known that we can prove local well-posedness of a few SPDE using a fixed point argument: Young's ...
mathex's user avatar
  • 399
0 votes
0 answers
104 views

Laplace transform of a stochastic process

Let $R := (R_1, R_2)$ be a two-dimensional diffusion process defined by the following SDE: $$\mathrm{d}R_{1,t} = -\lambda_1 R_{1,t} \, \mathrm{d}t + \lambda_1 \sigma(R_{1,t}, R_{2,t}) \, \mathrm{d}W_t$...
Greyearl's user avatar
2 votes
1 answer
244 views

If $u$ is harmonic, $\exists \alpha,\beta \in \mathbb{R},\forall x\in \mathbb{R}^d,u(x) \leq \alpha |x|+\beta,$ then $u$ is affine

We consider a harmonic function $u:\mathbb{R}^d \to \mathbb{R}$ $(\Delta u=0).$ Suppose that $$\exists \alpha,\beta \in \mathbb{R},\forall x\in \mathbb{R}^d,u(x)\leq \alpha |x|+\beta.$$ Therefore $u-u(...
mathex's user avatar
  • 399
0 votes
0 answers
86 views

Application of Ito's formula to Liouville's theorem

Liouville's theorem for bounded harmonic functions could be proved using Ito's formula, martingale convergence and Blumenthal's 0-1 law. I tried checking the classical books on Brownian motion and ...
mathex's user avatar
  • 399
2 votes
1 answer
135 views

Is every simply connected domain regular?

Recall that a domain $D \subseteq \mathbb C$ is called regular if for each point $x \in \partial D$, we have $\mathbf P_x\lbrack \tau_D = 0\rbrack = 1$, where $\tau_D = \inf\{t > 0 : B_t \notin D\}$...
Focus's user avatar
  • 75
3 votes
1 answer
308 views

Each diffusion SDE is associated to a *unique* family of transition kernels

I consider an SDE of the form $dX_t=b(X_t) \, dt + \sigma(X_t) \, dW_t$, with $b$ and $\sigma$ globally Lipschitz on $\mathbb{R}^n$. How can I prove that there exists a unique family of transition ...
No-one's user avatar
  • 1,037
3 votes
1 answer
155 views

Is a semimartingale that is continuous a continuous semimartingale?

Let $X$ be a centered semimartingale that has continuous sample paths almost surely. Is it then true that $X$ is a continuous semimartingale? Meaning that $X$ has a decomposition $X=M+V$ where $M$ is ...
user479223's user avatar
  • 1,302
1 vote
1 answer
156 views

Is deterministic evolution preserved under weak converge of stochastic processes?

Suppose you have a sequence of continuous stochastic processes $X_N$ with $X_N(0)=0$, and that $X_N$ converge weakly on the space of continuous functions, to a stochastic process $X$. Suppose $X_N$ ...
user479223's user avatar
  • 1,302
2 votes
0 answers
90 views

How much is known about the action functional for small noise diffusions with general volatility coefficients?

Let $W$ be a d-dimensional Brownian motion, and for every $\varepsilon > 0$, let $X^\varepsilon$ be the solution to the SDE $$dX^\varepsilon_t = b(X^\varepsilon_t) \, dt + \varepsilon \sigma (X^\...
Nate River's user avatar
  • 4,832
2 votes
0 answers
97 views

Bounding from below the distance between SDE started from different initial conditions

Let $W$ be a standard one dimensional Brownian motion, and let $X$ be the solution to the SDE $$dX_t = \mu(X_t) \, dt + \sigma(X_t) \, dW_t$$ with $\mu, \sigma: \mathbb R \to \mathbb R$ Lipschitz ...
Nate River's user avatar
  • 4,832
1 vote
0 answers
104 views

Lower bound of $\mathbb P[\sup_{t-\theta\le s\le t}|X_s-x|\le \varepsilon \mid X_t=x]$ (without observing history)

Let $X$ be the solution to some stochastic differential equation $$dX_t =b(X_t) \, dt+a(X_t) \, dW_t,\quad \forall t>0.$$ Here $b,a: \mathbb R^d \to\mathbb R^d$ are bounded and Lipschitz and $W$ ...
Fawen90's user avatar
  • 975
2 votes
1 answer
294 views

Stationary Distribution of Langevin Dynamics driven by Lévy Process

Let $f\geq 0$ be a Lipschitz function and let $(L_t)_{t\geq 0}$ be an $\alpha$-stable Lévy process ($0<\alpha<2$, possibly multivariate). Consider the process given by $$dX_t=-\nabla f(X_t)dt+\...
Small Deviation's user avatar
1 vote
0 answers
180 views

Characteristic function of stochastic integral of a pure jump Lévy process with respect to another pure jump Lévy process

(I am cross-posting this question here from MSE: https://math.stackexchange.com/questions/4725734/characteristic-function-of-stochastic-integral-of-a-pure-jump-l%c3%a9vy-process-with. I apologize if ...
Tom's user avatar
  • 11
2 votes
0 answers
114 views

Martingale regularization

Consider a submartingale $X,$ then for almost every $\omega \in \Omega,$ for every $v \in \mathbb{R},\lim_{u \in \mathbb{{Q},u \uparrow v}}X_u(\omega)$ exist in $\mathbb{R}.$ I was wondering if there ...
mathex's user avatar
  • 399
2 votes
1 answer
137 views

Gaussian Poincare inequality in $1$ dimensions together with localization issue

Let $d\mu$ be a Gaussian measure on $\mathbb{R}$ with the center $a \in \mathbb{R}$ and variance $1$. Let $B(a,r) \subset \mathbb{R}$ be the interval $[a-r,a+r]$. Then, for any smooth mapping $f : \...
Isaac's user avatar
  • 2,749
0 votes
1 answer
178 views

Characteristic function of quadratic variation of compound Poisson process

If I have a compound Poisson process whose characteristic function is known, is there a way to calculate the joint characteristic function of this process and its quadratic variation process? If not ...
Frimousse's user avatar
-1 votes
1 answer
137 views

joint density of two relevant random variables

It seems that for most of the examples to derive the joint density of two or more random variables, the random variables themselves need to be independent. Is it possible to get the joint density of ...
Wang Jing's user avatar
3 votes
0 answers
94 views

Dealing with noise that is white in time, colored in space numerically

I am broadly working on a dynamic process where we want to see how a field $\rho(r)$ changes in space in time with thermal noise. The system is biased around a thermodynamic saddle point dictated by $...
Yhtomit's user avatar
  • 31
1 vote
1 answer
90 views

Volterra Processes (integration wrt Brownian motion): reference request

I need some references about Volterra processes $Y=(Y_t)_{t\geq0}$ defined as $$ Y_t:=\int_{0}^{t} g(t,s)dB_s, \ \ t\geq 0,$$ where $B=\left(B_t\right)_{t\geq0}$ is a brownian motion and $g$ satisfies ...
Joegin 's user avatar
3 votes
1 answer
262 views

Strong blow up limits for SDE

Note: This is a strengthening of the following result, motivated by the need for strong convergence in applications. Let $W$ be a one dimensional standard Brownian motion, and let $X$ be the solution ...
Nate River's user avatar
  • 4,832
1 vote
0 answers
139 views

Eigenvalues/eigenfunctions of a diffusion generator

Consider the following symmetric second order diffusion operator, defined, for $\phi \in \mathcal{C}^{2,1}_c\left(\mathbb{R}\times \mathbb{R}_+\right)$, by: $$L\phi := \lambda_1 \partial_{R_1}(R_1 \...
Greyearl's user avatar
5 votes
1 answer
278 views

Does the entropy of a SDE with nondegenerate noise always increase?

Let $W$ be a standard Brownian motion, and let $X$ be the solution to the one dimensional SDE $$dX_t = \sigma(t, X_t) \, dW_t$$ with initial condition $X_0 = x_0$ a.s. for some $x_0 \in \mathbb R$. We ...
Nate River's user avatar
  • 4,832
2 votes
1 answer
326 views

Convergence of the quadratic variation process

Suppose we are given a sequence of stochastic processes $X^n, n\in\mathbb{N},$ with finite quadratic variations and a stochastic process $X$ such that for every $t\geq0$ $$ \lim_{n\to\infty}\mathbb{E}(...
El_mago's user avatar
  • 99
0 votes
0 answers
73 views

Calculation of the difference of two Brownian bridges

I was told that the difference of two independent brownian bridge process is $\sqrt{2}$ times a brownian bridge process, i.e., $$B_{1t} - B_{2t} = \sqrt{2}B_t$$ where $B_{1t}$ and $B_{2t}$ are ...
John Smith's user avatar
4 votes
0 answers
211 views

A notion of SDE via the martingale representation theorem

$\newcommand{\d}{\mathrm{d}}$It is well-known that differentiating stochastic processes with respect to time is usually impossible in the usual sense. For instance, a Brownian motion $W$ on a ...
Emily's user avatar
  • 10.7k
2 votes
0 answers
58 views

On a real smooth version of white noise distribution theory

In white noise analysis, one starts with a real Gelfand triple $\mathcal{N}\subset \mathcal{H} \subset \mathcal{N}^{*}$ and produces out of it, using complexifications along the way, the complex ...
S.Z.'s user avatar
  • 473
2 votes
1 answer
141 views

Local martingale with increasing process

Here is a problem in stochastic calculus: If $M_t$ is a continuous process and $A$ an increasing process, then $M$ is a local martingale with increasing process $A$ if and only if, for every $f\in C^2$...
Liu Wei's user avatar
  • 21
1 vote
0 answers
105 views

Modulus of "set"-continuity for Wiener Field

My question concerns some "set-wise" continuity properties of Gaussian random fields, more specifically of Wiener fields (see definition here: https://encyclopediaofmath.org/wiki/...
BabaUtah's user avatar
1 vote
1 answer
108 views

When is the probability measure on the "direct product" via the Kolmogorov extension theorem supported on the "direct sum"?

Let me restrict to the case of Hilbert spaces, which seem simplest. Let $\{H_n\}$ be a sequence of (possibly infinite dimensional) Hilbert spaces and $\{ \mu_n \}$ be a sequence of Borel probability ...
Isaac's user avatar
  • 2,749
2 votes
1 answer
333 views

Interacting particle system: how are the particles independent conditionally to the knowledge of their initial positions?

$\newcommand{\Ex}{\mathbb E}\newcommand{\diff}{\ \mathrm d}$Let $(\Omega, \mathcal F, \mathbb P)$ be a probability space. $B=(B^1, \ldots, B^N)$ independent one-dimensional Brownian motions. $X=(X_0^...
Akira's user avatar
  • 851
2 votes
0 answers
228 views

Ito lemma for SDEs on a Lie group

I'm trying to generalize the theorem described in this paper https://arxiv.org/abs/2001.01098 to the case of a semisimple compact matrix Lie group. In doing so i'm trying to define a formula ...
Marco's user avatar
  • 263
1 vote
2 answers
195 views

Converse Cameron-Martin theorem for shifts by adapted processes

Let $W$ be a standard one dimensional Brownian motion, $\mathcal F_t$ its natural filtration, and $\mathbb P$ be the induced Wiener measure on $\Omega := C[0, 1]$. Given a $C[0, 1] $ valued random ...
Nate River's user avatar
  • 4,832
3 votes
1 answer
130 views

Are there any known results on the probability distributions of perpetuities with power law discount rates?

Currently I am working on studying stochastic integrals of the form: $$Z_\infty = \int_0^\infty e^{-f(t)}\mathop{d}S_t$$ where $S_t$ is a Compound-Poisson process with Exponentially-distributed ...
jam jelly's user avatar
1 vote
2 answers
276 views

Convergence in sup norm of elementary integrals to the Itô integral process

Let $W$ be a standard one dimensional Brownian motion, and $X$ a continuous process adapted to $W$ such that $\int_0^T X^2 \, ds < \infty$ almost surely for some $T > 0$. Define for any sequence ...
Nate River's user avatar
  • 4,832
4 votes
1 answer
134 views

Finite number of ergodic random Dirac measures

Let $\Omega$ be a Polish locally compact space and $(\Omega, \mathscr{F}, \mathbb{P})$ be a probability space. Consider a measurable map \begin{align*} \theta\colon T\times \Omega &\to \Omega\\ (t,...
Eduardo's user avatar
  • 757
1 vote
1 answer
90 views

How to obtain this differential relation about moments of a stochastic process?

$\newcommand{\Ex}{\mathbb E}$ I'm reading an argument in the proof of Proposition 3.8. in the paper Nonlinear self-stabilizing processes - I Existence, invariant probability, propagation of chaos. ...
Akira's user avatar
  • 851
7 votes
1 answer
640 views

How is the Gronwall lemma used in this paper?

Let $(X_t, t \ge 0)$ be a $\mathbb R^d$-valued stochastic process. Let $\lambda>0$. Assume we have $\mathbb E [|X_0|^2] < \infty$ and $$ \mathbb E [|X_t|^2] - \mathbb E [|X_0|^2] \le -2 \lambda \...
Akira's user avatar
  • 851
2 votes
0 answers
241 views

KL Divergence between the solution to two SDEs

What is the KL divergence between the laws of solutions to SDEs? That is, let \begin{align*} dX^1&=b_1(X^1,t) \, dt+\sigma(X^1,t) \, dB\\ dX^2&=b_2(X^2,t) \, dt+\sigma(X^2,t) \, dB \end{align*}...
user499216's user avatar
1 vote
1 answer
109 views

Asymptotic expansion on the following integral of exponential function

I wish to obtain the asymptotic for the following integral $$ \int_{r: \|r\|\le 1} \exp(M\cdot a^Tr) \, dr, $$ where $a$ is a given vector in $\mathbb{R}^d$, $\|\cdot\|$ is a general norm function and ...
user497696's user avatar
1 vote
0 answers
95 views

Reference request: $d X_t = b(X_t) d t + f (p_t(X_t)) d W_t$ where $p_t$ is the p.d.f. of $X_t$

Let $b:\mathbb R^d \to \mathbb R^d$ and $\sigma:\mathbb R^d \to \mathcal M_{ d\times q} (\mathbb R)$ be Lipschitz. Let $(W_t, t\ge 0)$ be the standard $q$-dimensional Brownian motion. Then $$ d X_t = ...
Analyst's user avatar
  • 647
1 vote
1 answer
143 views

The space of linear operators between Hilbert spaces has martingale type 2

I am trying to prove whether the space $L(H,K)$ has martingale type 2 for Hilbert spaces $H,K$. It is known that Hilbert spaces have martingale type 2, so I was wondering whether the space of bounded ...
Shashi's user avatar
  • 113
3 votes
1 answer
322 views

Reference request for a Riemannian Fokker-Planck equation

The original post is in StackExchange but no one has answered it yet. I personally think it is more related to the research area so I put it in MathOverflow. Below is the question in the original post:...
Eddie Lin's user avatar
  • 187
4 votes
1 answer
168 views

Small noise limits with irregular drift

Let $W$ be a standard $d$-dimensional Brownian motion. Suppose $b: \mathbb R^d \to \mathbb R^d$ is measurable and bounded. Consider, for every $\varepsilon > 0$, the solution $X^\varepsilon$ on $[0,...
Nate River's user avatar
  • 4,832
1 vote
1 answer
100 views

How is $\mathbb E[ \int_0^T H_s^2 \mathrm d s] < \infty$ important for this claim?

Let $(\Omega, \mathcal F, \mathbb P)$ be a probability space. Let $H=(H_t, t\ge 0)$ be a stochastic process with continuous trajectories. Fix $T>0$. For $n \ge 1$, we define $$ H_{s,n} := \sum_{i=1}...
Analyst's user avatar
  • 647

1
2
3 4 5
19